Putnam Problem Set 11 November 3rd

You might also like

Download as pdf or txt
Download as pdf or txt
You are on page 1of 1

Putnam Problem Set 11

November 3rd

1. How many five-digit multiples of 3 end with the digit 6?

2. Describe the set of all points P in the plane such that exactly two tangent
lines to the curve y = x3 pass through P .

3. (P 1968 A1) Prove


1
x4 (1 − x)4
Z
22
−π = dx.
7 0 1 + x2

4. (P 2001 B3) For any positive integer n, let hni denote the closest integer

to n. Evaluate

X 2hni + 2−hni
.
2n
n=1

5. (P 1980 B4) Let A1 , A2 , . . . , A1066 be subsets of a finite set X such that


|Ai | > 12 |X| for 1 ≤ i ≤ 1066. Prove there exist ten elements x1 , . . . , x10
of X such that every Ai contains one of x1 , . . . , x10 . (Here |S| means the
number of elements in the set S.)

You might also like